You are on page 1of 28

Evidence Haikus: This is my outline Offered for the truth of the Matters asserted

Evidence Outline Professor Seigel, Fall 2004 Drew M. Altman

Who is this witness? I really like this jury Another beer please!

I. Preliminary Theory A. The law of evidence: a. Law of the courtroom (bench and jury trials) FRE 101 provides the scope and applicability of the federal rules. FRE 1101 provides those proceedings to which the rules do not apply. b. Do not apply to sentencing hearings, but judges still expect lawyers to act within the scope of the rules hearsay issues are excepted. c. FRE took over from the common law to provide a uniform system. B. Why do we have the Rules? a. Primary Reasons. 1. Pragmatism - Adversarial system if no rules on admission the parties to a suit would fight continually. Need to keep things relative short. 2. Accuracy in factfinding parties will shoot for emotion and we need the factfinder to decide based on truth. Note the tension between pragmatism and accuracy succinctness and truth dont always go hand-and-hand. 3. Mistrust of juries the FREs are designed to preserve the jury system and foster accurate outcomes notwithstanding the inexperienced fact-finders. Seigel thinks that a jury as a whole is generally worthy of trust. 4. Provides some trump-cards: e.g. spousal immunity. Tension between accuracy and social policy. 5. Further the substantive law e.g. burdens of proof. 6. Preserve trial as ritual and theater want citizens to see and accept what happens in the courtroom. 7. Dispute resolution FRE provide one means of going about this. C. Two Primary Rules a. Rule 104(a) The judge generally decides upon preliminary questions of admissibility of evidence. Interpretation is a matter for the jury who decides upon the weight of the evidence and the credibility of witnesses. b. Rule 104(b) conditional relevance (more on this later) when relevancy is dependent upon the fulfillment of a condition of fact, the court shall admit it subject to the introduction of evidence sufficient to support a finding of the fulfillment of the condition. II. Relevance A. Preliminary Matters a. A relational concept relevance carries meaning only in context. Defined in part by substantive law and partly by issues raised by the parties. 1. Criminal cases issues are raised by the indictment and Ds plea. 2. Civil cases issues raised by pleadings, and are refined through discovery and motions narrowed by pretrial conference and order. b. FRE 401 Evidence is relevant if it has any tendency (probativeness) to make the existence of any consequential fact (materiality) more or less probable. Relevancy argument is over admissibility! Evidence need not be sufficient to be admissible. any tendency is taken w/grain of salt. E.G., if there is a study showing that UF alums are more likely to speed, and the D in an accident case is a UF alum, should this come in? Most judges would say no this evidence does not come from general experience, and, if a piece of evidence has so little a tendency to support a premise, it may not be admitted by the judge. c. FRE 402 Relevant evidence is generally admissible and irrelevant evidence is not. d. Direct vs. Circumstantial Evidence: 1. Direct evidence that which, if accepted as true, necessarily establishes the point for which it is offered. (e.g., E says, I was loading the truck for the company with which I am employed.) 2. Circumstantial evidence even if fully credited, may still fail to support (let alone establish) the point in question because an alternative explanation seems as probable or more so. Requires inferences before it will support a given point in question. CE involves a chain of inferences. (e.g., F says, E is an employee of the company because I say him loading one of the companys trucks.) 3. Rules do not draw a distinction between the two, and the latter is not necessarily inferior to the former. e. Rationality evidence law, particularly relevance, emphasizes reason and logic over psychology or emotion. f. Backward looking everyday decisions look forward, but courts determine historical fact. 1

g.

Proof At Trial: Ultimate Issue(s) I Ultimate Issue(s) II (legal facts) Criminal Guilty or Not Guilty Mens Rea Actus Reus Attendant Circumstances Defenses D shot victim in the heart spent days planning Bullet hole through heart. Fingerprints on gun. Receipt for ski mask. D said, its all my fault. Civil 1. Liable or not; 2. Damages or not E.G. Negligence, Proximate cause Breach of K Harm, etc. D failed to stop at red light. D failed to pay for services rendered D said, oh no, it wasnt green, at scene. Stop-payment order found in bank records. D said, its all my fault.

Facts of Consequence (Prove/disprove, either directly or through inference, ultimate issues) Subsidiary Facts 1. Through inference prove or disprove facts of consequence. 2. Through inference prove or disprove ultimate issues

B. Logical Relevance & Establishing Relevance a. Modern approach: relevance under the FREs combine materiality and relevance into one concept. b. Evidential hypothesis party offering evidence must have an EH to support the relevance of the evidence. 1. General premise in the EH proposition of general knowledge about the ways of the world or human nature. 2. Specific premise at least one in the EG. 3. Conclusion the EH sets out the conclusion toward which the evidence points. 4. EH by induction generalization, the way we usually think. Premise does not necessarily lead to a given conclusion but supports it nonetheless. - E.g., problem 2-A: People who drive red sports cars are more likely to speed (major premise). D drove a red sports car (minor premise). D was probably speeding (conclusion). - Fact of consequence D was speeding at the time of the accident. - UI II: Speeding = Negligence. - UI I: Negligence = liability. 5. EH by deduction where the stated premise necessarily leads to a given conclusion (standard syllogism). (e.g. humans are mortal (major premise.) Socrates is human (minor premise). Socrates is therefore mortal (conclusion)). c. Attack on EH: the more general and minor premises the EH has, the easier it is to attack. Also, the longer the chain of inference required, the easier it is to attack. d. Evidence of flight: Generally admissible (see problem 2-B). EH those who flee are more likely to have committed a crime (major premise). D fled (minor premise). D is therefore more likely to have committed a crime (conclusion). Flight is usually a subsidiary fact, but it goes directly to the ultimate issue of guilt. C. Pragmatic Relevance a. Prejudice and confusion. i. Rule 403 Confers upon the judge broad discretion to exclude evidence for 1. Unfair prejudice; 2. Confusion of the issues; 3. Misleading the jury; 4. Considerations of undue delay; 5. Waste of time; 6. Needless presentation of cumulative evidence (Holmes: a concession to the shortness of life.) ii. 403 does favor admissibility: only when probative value is substantially outweighed by one of the other factors will evidence be excluded. iii. Stipulations where parties stipulate to given point, any further evidence on that point will have seriously reduced probative value.

iv. v. vi.

i. ii. iii. iv. v. vi. i. ii. iii. iv.

i. ii. iii.

i. ii.

State v. Chapple D stipulates that Dee shot the victim in the head. Disgusting photos of murder victim, while relevant on the elements of proving murder, should have been excluded from evidence because they were purely inflammatory and did not bear on the only uncontested issue, which was whether D was in fact Dee. Old Chief in drug possession case, evidence of Ds prior drug convictions excluded as too prejudicial. Problem 2-E Exploding Gas tank. Strict product liability suit (Pinto-like situation). D wants to introduce evidence that car causing the accident 1) was travelling at 68 mph; and 2) that the driver pled guilty to involuntary manslaughter. Both are relevant on the issues of foreseeability and cause (fact of consequence and UI II, respectively), but only #1 is admissible. #2 is prejudicial: confusion of the issues the jury may not understand that two persons/entities may be separately liable for the same event. b. Limited Admissibility Confining the Impact of Proof FRE 105 Limiting Instruction. Used when evidence has a spillover effect: part is relevant, and part is either irrelevant or barred by 403 (or some other rule e.g. evidence of liability insurance under FRE 411). Also whenever evidence is admissible for one purpose and not another (e.g. an o/o/c statement admitted to show effect on listener but not admissible for truth of the matter asserted), a limiting instruction is available. 105 operates upon request often has the undesired opposite effect of calling attention to the portion of the evidence that one wants the jury to disregard. LIs are often thought to be ineffective. Catch 22 failure to object usually proscribes that issue from being part of an appeal. If redaction is possible, 105 contemplates its use. c. Completeness providing context FRE 106 when a writing or recorded statement is introduced, the adverse party may require introduction of any other part (or any other writing or recorded statement) that should in fairness be considered with the proffered statement. Although 106 speaks only to recorded or written statements, one may object to other evidence upon the spirit of 106; thus, additional parts of a conversation may need to be admitted. 403 may also be used to balance, and admit or exclude the whole accordingly. E.g., military plane crash and subsequent suit claiming that mechanical malfunction was cause. D submits a portion of a letter written by decedents husband that cause of accident was pilots fault, but omits portions of the letter that ultimately attribute fault to D. In rebuttal, P asks about other portions of the letter by invoking 106. 1. Interrupt function permits counsel to object immediately and require other statements to be read contemporaneously (not done in this hypo). 2. Rebuttal function though 106 reads, at that time, adverse parties may answer an incomplete presentation later in trial (as in hypo). ACNs support this. 3. Trump function 106 may trump rules such as hearsay. In the hypo, D used the admissions doctrine to admit Ps written statements. Although one may not introduce ones own o/o/c statements, P may do so here (other portions of his own letter) because the rule of completeness requires that they be introduced. A party trying to be sneaky can really get screwed if the other side invokes 106! d. Relevance Rule 104(a) vs. 104(b) Simple relevance Whether a particular point, which proffered evidence concededly tends to establish or refute, is consequential per 401 is solely for the judge to decide. Weight for the jury to decide. 104(e). There is a continuum, however, between simple relevance (admissibility) and weight. Conditional Relevance 104(b) when relevance turns on the fulfillment of a condition of fact, the judge performs only a screening function: When different answers are reasonable, the jury decides. Evidence is admitted upon, or subject to introduction of enough other evidence sufficient to support a finding. 1. e.g., a spoken statement is used to prove notice to X, but it is without probative value unless X heard it. Thus, relevance of the statement turns on satisfying the condition of proving that X heard it. 2. The judge basically has to decide whether or not a reasonable juror would be convinced that the condition is met. 3. E.g. 2-H: faulty bicycle brake allegedly cause of accident. Brake does not work now, but evidence is only relevant if it can be shown that brake did not work two years ago at the time of the accident. Can ask for conditional admission, but if expert cannot testify that two years of sitting in a dusty garage caused no change, the case is sunk (and the judge will be pissed if you asked for more time and failed to lay sufficient foundation for admission). e. Probabilistic Analysis and Statistics Conjunction where testimony by two witnesses must be accepted on a given point. The relevance of what each says might be said to be conditional on what the other says. Product Rule Where two or more facts together are needed to establish a point, the probability of the occurrence of each fact must be multiplied with one another. The product is the probability of the point that is sought to be established. 3

iii. iv.

403 objection where statistics are used, confusion of the issues may require their exclusion. People v. Collins made up statistics by prosecutor (including his statement to disregard reasonable doubt and focus only on the numbers) as to probability of a there being a black man with a blonde woman in a yellow Cadillac infected the case with fatal error and distorted the jurys traditional role of determining guilt or innocence according to long-settled rules. Problems in this case: 1. factors used by prosecutor had no foundation; 2. not mutually independent; 3. no witness saw the incident at issue directly, thus if any witness was wrong about what they saw (e.g. maybe a dark-skinned Latino rather than a black man), the stats had no relevance whatsoever. 4. Hard variables (stats) too convincing and dwarf the soft variables (testimony, other non-quantifiable evidence). v. Where statistics work DNA evidence and paternity cases. Indeed, statistical evidence is permitted in the courtroom. vi. The Blue Bus and naked statistics 80% of the buses that travel the route are Blue Bus Company Buses 1. Admissible under 401(a)? Yes. 2. This is naked statistical evidence, meaning that there is no other evidence on a fact of consequence or ultimate issue (in this case, both the FOC and UI is identity). 3. Best argument against admission: 403 without more, this evidence is misleading. 4. Second best argument: 104(b) a foundation must be laid to prove the statistic correct (if this id done, however, the objection is shot too). 5. Majority rule naked statistical evidence is insufficient to support a verdict. 6. Nessin if we permit naked statistical evidence to be sufficient, people will model behavior purely on market share (i.e., if 80% probability is enough to convict BB Co. as the liable party, then all those with high market share will either 1) reduce market share to reduce liability, or 2) simply act however they want since theyll get nailed for the wrongdoing of others anyway). III. Hearsay A. The General Rule a. Defined: An out of court statement offered to prove the truth of the matter asserted. (hearsay is a definitional test). b. Includes witness own o/o/c statement! What did you tell the officer? (objection). Rephrase, please tell the court what you saw. (no objection). c. 801 Defines hearsay and includes in sections (d)(1) and (d)(2) certain hearsay statements that are not h/s (exemptions) for purposes of the rule i. (d)(1) includes prior statements by witnesses (change from common law). ii. (d)(2) includes the admissions doctrine (preserved from common law). d. 802 provides that hearsay is inadmissible except as otherwise provided by the Rules. e. Hearsay risks: 1. Misperception relates to the declarants perception, who may or may not be the person on the witness stand; 2. Memory relates to the declarants memory. An o/o/c statement may be tainted by faulty memory; 3. Misstatement (risk of ambiguity or faulty narration.) simply, the o/o/c statement could have been uttered incorrectly from the declarants point of view. A person being facetious is a good example of ambiguity. 4. Distortion/deception whether conscious or unconscious the risk of lying, or more gently, the risk of insincerity or lack of candor. f. Ameliorating the risks in court: 1. Oath not only a form of moral suasion, but also implicates perjury risks. 2. Demeanor evidence Seigel says that this is often overestimated, but demeanor is important to evaluate a witness sincerity and veracity; 3. Cross examination this is the most important control, and the key reason why hearsay is generally inadmissible. g. Who decides? 1. The judge decides whether a statement is hearsay under Rule 104(a). 2. If a hearsay objection is not made in a timely manner, the issue is waived. B. What is a Statement a. 801(a) provides that a statement is 1. An oral or written assertion or 2. Nonverbal conduct of a person, if it is intended by the person as an assertion. 4

b. Conduct: 1. Assertive Conduct Conduct intended as an assertion (hearsay). E.g. nodding your head in response to a question. 2. Non-Assertive Conduct Not hearsay. E.g. a sea captain examining a ship and departing upon it with his family as evidence that the ship was seaworthy. The captain sought to assert nothing; thus not hearsay. Although some of the risks are still implicated, the risk of insincerity is significantly reduced. Also, a truck pulling into an intersection as evidence that the light had changed is not hearsay since the driver did not intend to assert anything by his conduct. i. Note: the risk of ambiguity still runs high with non-assertive conduct, but the federal rules were drafted in favor of allowing non-assertive conduct to fall outside the h/s definition. This is predominantly due to the reduction in risk of insincerity/deception. ii. The Rules contemplate an assertion-focused definition of hearsay; thus, if there is no assertion, there is no hearsay. The competing definition is declarant-focused and that could turn non-assertive conduct into hearsay because such conduct still depends on the credibility of the declarant. c. Non-Assertive conduct as an Unintended Implied Assertion: 1. Like the letters in Wright v. Doe d. Tatham offered to prove that Wright was competent as a testator. May be analyzed as non-assertive conduct, but may be looked at as an unintended implied assertion. 2. If viewed as the latter, we move to the other side if Seigels chart, and we must decide if the unintended assertion is trustworthy. If it is, then it is not hearsay, otherwise, it is. The letters in Wright have a strong performative aspect that leads us to the conclusion that they are not hearsay. 3. The line between non-assertive conduct and unintended implied assertion seems to blur when we have pure conduct (e.g. pulling into an intersection) and conduct mixed with verbiage (e.g. mailing the detailed letters in Wright). Where there is verbiage, analyze on the r/h side of the chart (i.e. do not analyze as non-assertive conduct). d. Hearsay utilizes an intent-based definition 1. If the declarant intended to make an assertion, the statement is hearsay. E.g. its raining cats and dogs offered to prove that it was raining. 2. If there is no intent to make an assertion, there is no hearsay. E.g. someone calls a bookies illegal operation and states put $50 on horse #2 at Belmont if offered to prove that the place was being used for gambling: the caller did not intend to assert that the place he was calling was being used for illegal betting. 3. SILENCE Under the FRE, silence is not hearsay because it is non-assertive conduct. People generally do not try to deceive through silence. i. Cain v. George evidence of other guests who have stayed in a particular room not complaining (i.e. remaining silent) offered to prove that the heater had been working correctly. ii. If comment cards had been filled out by guests without any mention of a bad heater, wed analyze this on the r/h side of the chart as an unintended implied assertion: will still be not h/s since there is a strong performative aspect that would be made even more trustworthy by multiple declarants. C. When a Statement is Not Hearsay a. Context / Indirect hearsay U.S. v. Check error to permit investigating officer to give only his side of the conversation when it was clearly an attempt to incorporate the statements of an information, who would not testify at trial. 1. Sometimes admissible may be used for context to move the story along. 2. E.g., Officer, what did you do on 10/14/99? I had a conversation with informant. Without telling me what he said, what did you do next? I went to 34th and main and set up a stakeout. (objection, hearsay). Offer of proof: officer says that he saw D sell drugs at the stakeout location and arrested D and the buyer. The indirect hearsay (the informant obviously told the officer where to go) is permissible because it merely moves the story along. b. Impeachment: 1. Where offered for impeachment, a statement is not hearsay. 2. E.g. the bystander in court testifies, blue car ran red light. On cross, didnt you say blue car had green light? (objection). This is allowed in for the purpose of reducing the witness credibility. However, if this is the only evidence that the blue car did in fact have the green light, thats a problem: since offered for impeachment, there would be no substantive evidence on the issue. c. Verbal acts. 1. Defined: words, which by their utterance, change the legal position of the declarant. 2. We dont care about the truth value of these statements, only that the statement was made. We dont care what the words say, only what they do. 5

3. E.g. bank robber gives a note to the teller that reads, give me all of your money. If offered, might raise a hearsay objection. You may counter that it is not being offered for the truth of the statement, only that it has changed the legal position of the declarant from law-abiding citizen to a bank-robber. 4. E.g. I accept your offer changed the legal position of the declarant: now has a binding K. - Note: dont get tripped up when a verbal act also looks very much like it is being offered for the truth of the matter asserted. If a statement is a verbal act (or effect on listener), it may be offered for that purpose notwithstanding the dual implication. (e.g., as proof that tenant G terminated his month-to-month tenancy effective November 1, evidence that G sent owner H a letter in September reading, October will be my last month as tenant. I am vacating by November 1st. This is a verbal act). 5. Substantive law need to know this to spot a verbal act. d. Effect on Listener 1. Defined: a statement offered only to show its effect on the state of mind of the person who heard it (or read it). 2. May also be used to show a change in legal position of the person hearing the statement (e.g. agency). 3. Often used to prove reasonableness and notice: i. To prove it was reasonable to take Joe to the location of the gas leak, the statement Im Joe from the gas company. The statement is not being used to prove agency; thus, it is not offered for the t.o.m.a. and is not hearsay. ii. To prove that P had notice that the prescription medicine caused drowsiness, offering the label on the bottle that reads this medicine may cause drowsiness. Since not offered to prove that the Rx may in fact cause drowsiness, it is not h/s. e. Verbal Objects and Verbal Markers 1. Objects are considered trustworthy by nature. 2. Verbiage on an object is the o/o/c statement. 3. Pure verbal object where only the witness need look at the object to make the connection. Not hearsay. (e.g. matchbook found in Ds possession. Owner of restaurant called to the stand, and handed matchbook. If, without reading what the matchbook says aloud to the jury, the witness says, these came from my restaurant, the D is now connected to that location). Pure verbal objects are on the left side of the chart. 4. Impure verbal object where the evidence must be read/shown to the jury (e.g. the words Mildreds Big City Food on the matchbook). Detective on the stand actually reads the words on the matchbook in order to connect the D to having been at Mildreds. Offered for an indirect implied assertion (the matchbook in effect saying, Hi, Im from Mildreds!) on r/h side of chart. Still not h/s because of inherent trustworthiness. 5. Lack of deception the principal reason for admitting verbal objections into evidence. 6. Verbal Marker requires two witnesses. The assertive conduct at issue is that of one witness pointing out or otherwise marking something (e.g. a waitress pointing to the D in a restaurant months before trial). This testimony may be put together with a cops testimony identifying the person pointed out by the waitress. f. Circumstantial Evidence of State of Mind 1. If the Declarants state of mind is a material issue, a statement that manifests that state of mind is relevant. 2. E.G. in a wrongful death suit brought by husband, a statement by deceased wife in her will that husband is very cruel to me. If offered to prove that H was cruel, it is hearsay. However, if offered to show what wife thought of husband, which would tend toward proof of lower damages, the statement may be admitted. g. Circumstantial Evidence of Memory 1. Theory: the mind contains a snapshot that is relevant to something in the case. 2. The papier-mache man is the example used in the book. The officer testifying to the girls statement to him regarding the paper mache man is relevant to prove that the girl had been in that room; and, if the prosecution lays the foundation that the girl had no other way of getting such information, it will come in. The jury need not believe what the room looked like, only that it is evidence that the girl had such knowledge. h. The Borderland and Statements with Performative Aspects 1. ACN to 801: [V]erbal conduct which is assertive but offered as a basis for inferring something other than the matter asserted [is] also excluded from the definition of hearsay by the language of subdivision (c). 2. E.g. mailing of an eviction letter by a LL to A and B, 600 Main. If offered to prove that B lived at 600 main, it is hearsay. - What if offered to prove that A lived with B? May be analyzed as an implied assertion. The problem is whether intended or not. If the latter, then probably not h/s due to performative aspect. - Probably best to analyze as a verbal act eviction is a legal act. 3. E.g. W lies to police my husband is in Denver. Not being offered for the truth of the statement being offered for the relevant connection that wife who lies as to her husbands whereabouts makes it more probable 6

that H did something wrong. We have an unintended implied assertion and the performative aspect lying to a cop also goes to the trustworthiness (ironic the lie is trustworthy). Lying to an officer is also a crime; thus we have a verbal act as well. 4. Unintended Implied assertion Summary: If trustworthy: not h/s. Trustworthiness is increased by 1) strong performative aspects (put your money where your mouth is) and 2) multiple declarants. The reason for the statement being offered must be relevant! - EXAM NOTE: based upon my class notes, it appears that, wherever there is verbiage, it is best to argue that something is an unintended implied assertion before going down the non-assertive conduct route. (e.g. like the hotel case: if guests filled out comment card, do not argue non-assertive conduct since they didnt mention bad heater. Instead, cite unintended implied assertion) - Where verbiage and conduct are combined, you need to use verbiage analysis (e.g. as proof that X is an honest guy, evidence that he was given too much change at the register, and he handed is back and said, I think youve made a mistake here. Conduct (handing money back) combined with verbiage. Not h/s because unintended implied assertion and he put his $ where his mouth is) D. Hearsay Exemptions (not h/s) Part 1: 801(d)(1) a. Theory behind admissibility of all exceptions and exemptions is that of reliability (one exception: admissions) Where that is insufficient, we require necessity (i.e. for all of the 804 exceptions). b. 801(d): 1. (d)(1) three exemptions for prior statements. 2. (d)(2) The admissions doctrine. Five exemptions (see section E, infra). c. 801(d)(1)(A) Prior Inconsistent Statements three requirements for admission (reliability is the key): 1. Declarant must now be subject to cross examination concerning the prior statement; i. If witness says I dont remember, you have to decide whether witness is feigning ignorance or not. If the former, then witness is still subject to cross examination on the statement. If the latter, there is a split (Owens battered prison guard case Supreme Court stated that one is still subject to cross exam even if memory is bad enough to make the person unavailable for purposes of Rule 804 this was a (d)(1)(C) matter and, since the guard had some memory, the Courts statement is dicta). ii. Where the statement at issue is testimonial, you may have a Crawford issue, because the witness must be effectively subject to cross-examination regarding testimonial statements (confrontation clause). 2. Statement must be inconsistent with prior testimony; and i. inconsistent does not mean diametrically opposed. 3. Prior statement must have been made under oath in a proceeding or deposition. i. This requirement is not taken lightly: a stationhouse statement is rarely if ever admissible (State v. Smith is a rogue case). ii. Depositions are particularly reliable since both sides are present, cross examination is permitted, and an oath is administered. iii. Grand jury testimony, though ex parte, is admissible under 801(d)(1)(A). The solemnity of the setting plus the oath makes such testimony quite reliable. iv. A preliminary hearing is clearly a proceeding (cross exam is allowed). v. This exception works well with turncoat witnesses. d. 801(d)(1)(C) Prior Statements of Identification 1. Exception: one of identification of a person made after perceiving the person 2. Theory is that o/o/c statements of identity are often more trustworthy because they are i. Often made closer to the actual event; ii. Not subject to leading questions often asked at trial; and iii. Witness is still subject to cross examination at trial. 3. Possibility of double hearsay (permitted subject to rule 805): i. State v. Motta objection to use of composite sketch for identity. ii. First statement: declarant to sketch artist (one of identity and is admissible); but the Second statement the sketch artists rendition (statement need not be verbal) does not fit the exemption (he did not perceive the person). iii. However, the witness actually made two statements: 1) declarant to sketch artists; and 2) she looked at the rendition and said, thats him. Both statements are admissible under 801(d)(1)(C). She has effectively adopted the sketch artists statement (the sketch) as her own. 4. What if witness cannot recall identifying the D? (split) i. Owens cross exam requirement satisfied despite assertion of memory loss. 7

ii. Crawford Under confrontation clause, testimonial hearsay, including statements give to police, cannot be admitted unless the accused has a chance to cross-examine. E. Hearsay Exemptions Part 2: 801(d)(2) the Admissions doctrine. a. Admissions by party-opponent come in for the truth of the matter(s) they assert; and they represent the one exception to the reliability requirement inherent in all exemptions and exceptions. b. Theory: the adversarial system. 1) each party is responsible for making or breaking, winning or losing his/her own lawsuit by his/her conduct both in and out of court; and 2) you cant really bitch about being unable to cross examine yourself! c. Individual admissions 801(d)(2)(A): the statement is offered against a party and is (a) the partys own statement, in either an individual or a representative capacity. No against interest requirement statements by a party are broadly admissible against him. No need for first hand knowledge: if the party said it, hes stuck with it (e.g. in dog bite case where owner did not see bite occur but admits o/o/c that his dog bit the victim). Admissions are party specific! Where you have co-Ds, you cannot offer D1s admission against D2. Sleeping declarant statements usually kept out. Drunk/injured declarant statements usually let in. However, where a declarant is severely injured or incapacitated at the time of the statement, courts will often exclude. Involuntary admissions barred by the 5th Amendment. A severely injured or drugged D may be deemed to have admitted something involuntarily under those circumstances. Nolo and guilty pleas subsequently withdrawn not admissible (precluded by 410(2)). d. The Bruton issue: (applies only in the criminal context) This is a Constitutional issue involving admissions by one D implicating a co-D. Brutons coconspirator, Evans, admitted to a postal inspector (paraphrasing), Bruton and I did it. A limiting instruction was given that the statement was not to be used against Bruton, but he was convicted along with Evans. Supreme Court reversed, holding that such testimony is too damaging (prejudicial) and usually cannot be repaired, notwithstanding a limiting instruction. Confrontation clause: D1s confession implicating D2 is analogous to D1 accusing D2; thus, D2 has the right to confront D1 has his accuser. However, D1 has the 5th Amendment right not to take the stand, which effectively strips D2s right to confront his accuser. This is the quintessential Bruton rule problem. Curing the problem: (1) Redaction may work depending on the form of the statement. (2) Separate the cases last choice, but most effective and done where no other solution works. (3) Exclude the admission. (4) Two juries done by progressive courts, but you run the risk of conflicting verdicts. Richardson v. March (S.Ct) (admitting confession by W describing events culminating in robbery and murder over codefendant Ms Bruton objection: Ws confession only incriminated M when linked with evidence introduced later in the form of Ms own testimony when such linkage is necessary, it is a less valid generalization that the jury will not likely obey the instruction to disregard the evidence) - Basically, if the jury has to make a logical connection between the factual information and the statement, this is good enough; and the line has been drawn here. e. Adoptive Admissions 801(d)(2)(B) a statement of which the party has manifested an adoption or belief in its truth If a cop asks, were you speeding? and you reply yes, the cops question will have been adopted by you in your response. If not, we have the absurd situation where we admit only the word yes without any context. This process is called attribution. Example (H & W who were embezzling): H at dinner party with W next to him. Says to another guest, hey, you can run a great scam by skimming money from the business and giving the receipts to the accountant. If you get caught, blame the accountant! W says, yea, and were going to Paris on the money! - W has clearly adopted the truth of what H was saying. Indeed, she added to it. - If W had remained silent, or said my drink is empty and walked away, there is much more of a question. Rules for tacit admissions: At a minimum, tacit admissions should be made to appear that: (1) the party heard the statement; (2) the matter asserted was within his knowledge, and, perhaps most importantly; (3) the occasion and nature of the statement were such that he would likely have replied if he did not mean to accept what was said notwithstanding the foregoing, the statement should be excluded if it appears that; 8

i. ii. iii. iv. v. vi. vii. i. ii. iii. iv. v.

vi.

i. ii. iii. iv.

v.

i. ii. iii.

iv.

i. ii. iii. iv. v. vi. vii. i. ii.

iii. iv.

(4) the party did not understand the statement or its significance; (5) some physical or psychological factor explains the lack of reply; (6) the speaker was someone whom the party would likely ignore; or (7) the silence was in response to questioning or comments by law enforcement during interrogation after Miranda warnings have been, or should have been, given. f. Admissions by Speaking Agents: 801(d)(2)(C) Statements by a person authorized by a party to speak for it. Theory: when a person hires another to speak for him, it is fair to allow the words of the latter to establish facts at trial against the former. This will likely apply to attorneys, partners, and corporate officers (not the same as those agents who do not have speaking authority like the truck driver for your company, that rule is 801(d)(2)(D)). Sometimes, (d)(2)(C) statements are verbal acts and hearsay analysis isnt even required. - E.G. real estate broker selling a barn for the owner says theres a tractor in the barn thats included in the price. If used to prove that the tractor in the barn is included in the price that is a verbal act, and is not hearsay. If used to prove that there is actually a tractor in the barn that is hearsay, and 801(d)(2)(C) will make it not h/s and get it in. Pleadings from prior lawsuits are usually admissible against the party who filed them. (But admissions in pleadings may not be used in subsequent actions given FRCP 36(b)) Interrogatories also usually allowed in. Expert testimony inadmissible against the party who hired them not presumed to be within the partys control. Not sufficient by itself! 801(d)(2) provides that The contents of the statement shall be considered but are not alone sufficient o establish the declarants authority under subdivision (C). . . . Thus, if the statement is the only evidence on the agency relationship, that relationship has not been adequately proven. g. Admissions by Employees and Agents: 801(d)(2)(D) A statement by the partys agent or servant concerning 1) a matter within the scope of the agency or employment, 2) made during the existence of the relationship. The contents of the statement alone are not sufficient to establish the agency or employment relationship and scope thereof. There is no personal knowledge requirement. (Like with the wolf that theoretically bit the child: Mr. Poos statement that Sophie had bit a child that day is admissible notwithstanding his lack of personal knowledge). Statements of the principal are inadmissible against the agent. Employer cannot prevent admission by arguing that the agent was not authorized or hired to make stupid or damaging statements tough shit, the statements come in. Meeting notes and other internal books or records ACNs address this actually in (d)(2)(C): someone was authorized by the company to create these and they are generally usable against the principle without regard to any intent to disclose to third persons. Of course, 403 always applies. E-mail if contents within the scope of employment, it is admissible. h. Coconspirator Statements: 801(d)(2)(E). A statement by a coconspirator of a party 1) during the course of and 2) in furtherance of the conspiracy. Three elements: (1) declarant and D conspired (coventurer requirement) and statement made; (2) during the course of the venture (pendency requirement); and (3) in furtherance thereof (furtherance requirement). Admissible in civil and criminal trials whether or not they involve a conspiracy charge. Implication of Bourjaily v. U.S. - The proffered statement alone is not sufficient to establish the existence of the conspiracy and the participation therein of the declarant and the party against whom the statement is offered. - Because you cannot bootstrap the prevailing view is the following (U.S. v. Gambino): While the statement itself may be considered in deciding upon its admissibility, it must be supported by some other independent corroborative evidence (may be circumstantial) of a Ds conspiratorial conduct before the coconspirator hearsay can satisfy the preponderance standard (the general standard under 104(a)).

v.

Rationale: alleged coconspirator statements by themselves are insufficient to support a finding of conspiracy because they could be fabrications. Im with Cheech could have been a lie used only to puff the mobsters up. Order of proof it is generally preferred for the circumstantial proof of conspiracy to have been admitted before the coconspirator statement. This isnt required, and the statement may come in before the proof; but if the prosecution fails to later provide the evidence giving support to the statement, a mistrial may be required. Keep an eye out for Bruton concerns. Also, statements by undercover agents will not fit the exception since they dont meet the furtherance requirement; but they are at least admissible for context. F. Hearsay Exceptions FRE 803 a. Introduction Declarants availability or lack thereof is immaterial for purposes of admissibility under 803. Firsthand knowledge is explicitly required by a number of the 803 exceptions. Where silent, however, firsthand knowledge is still generally required. Most exceptions are based upon a circumstantial theory of reliability coupled with a necessity or practical convenience argument. Generally, if one of the hearsay risks (sincerity, perception, narration, or memory) is seriously reduced or eliminated, there is support for an exception. b. Present Sense Impressions 803(1) A statement describing or explaining an event or condition made while the declarant was perceiving the event or condition, or immediately thereafter. Immediacy is the key to present sense impressions: Declarant describes what he sees/feels as it occurs. A small lapse of a few minutes is ok, but nothing more. The statement must describe or explain an event or condition this underlies the theory of reliability under which the statements come in: lack of time to fabricate. One of the res gestae exceptions things that happened the relationship between the event and the statement were so intertwined that the event actually impelled the words out of the declarant. c. Excited Utterance 803(2) Requires: 1) a startling event or condition; 2) a statement relating to that event or condition; 3) made by a declarant with firsthand knowledge; 4) while the declarant was under the stress of excitement caused by the event or condition. As with 803(1), immediacy is the key. Indeed, both exceptions tend to overlap. (same theory, lack of time to fabricate). However, excited utterances may be much more delayed than PSIs. Declarant may be a bystander to the event at issue. It is generally accepted that kids stay excited for longer periods of time, thus the immediacy requirement may be relaxed somewhat one hour tends to be the limit. d. Then existing mental, emotional, or physical condition 803(3) Also one of the res gestae exceptions. Under the exception, hearsay evidence is admissible if it bears on the state of mind of the declarant and if that state of mind is an issue in the case. Theory: they present no memory or perception problems, and are thought to be necessary since they provide a glimpse inside a persons head. Four uses: 1. Prove declarants then-existing physical condition; 2. His then-existing mental or emotional condition; 3. His later conduct; 4. And facts about his will. Reaches only present state of mind: what declarant says on Wednesday about his mental state on Monday does not fit the exception. Cannot be used to prove the fact remembered or believed. (fact laden statements may bear on state of mind, but the facts recited cannot be proven through the hearsay statements). Example: in an extortion case, D says hes going to kill me if I dont pay up. - If offered to prove declarants fear (victims fear is an element in extortion), 803(3) allows the statement in. - Note, too, that it is also not h/s since it could be entered as circumstantial evidence of state of mind.1
1

i. ii. iii. iv.

i. ii. iii. iv.

i. ii. iii. iv.

i. ii. iii.

iv. v. vi.

Exam note: there is a lot of overlap between 803(3) to prove state of mind and circumstantial evidence of state of mind. Some statements will fit into multiple categories and full credit is awarded on the exam for all. 10

vii.

i. ii. iii. iv. v. vi.

i. ii. iii. iv.

- If offered to prove that D made the threat: still h/s. 803(3) does not permit the statement of memory or belief to prove the fact remembered or believed. - If statement were Im scared to death of D. 803(3) would still permit admission to prove fear; but it wouldnt come in as not h/s since the evidence is no longer circumstantial, but is instead direct. (still hearsay as to whether D made a threat). Proving Future Conduct through State of Mind: The Hillmon doctrine. - Declarant, Walters, writes a letter that he intends to go to Crooked Creek with Hillmon. (Walters is allegedly shot by Hillmon and an accomplice while at Crooked Creek; thus, he is unavailable death has a way of doing that . . . ) - Narrow Hillmon If offered to prove that Walters (just him) did in fact go to Crooked Creek: clearly admissible under 803(3). - Admissible to prove that Hillmon went to Crooked Creek? Depends! Broad Hillmon permits admissibility to prove the future conduct of third parties. Criticism this is a back door way of proving facts remembered or believed by Walters (i.e. something happened for Walters to believe that Hillmon was going to Crooked Creek probably a simple conversation. This conversation, if otherwise proffered, would be clear hearsay). Also if Walters wrote: Im going to Crooked Creek, and Hillmon told me hes going there too. Classic hearsay to admit the statement to prove that Hillmon went to Crooked Creek; thus, why should the statement come in merely because it is worded differently? U.S. v. Pheaster Takes the broad view of Hillmon. House Report suggests the narrow view. Split ultimately, most courts feel free to choose whichever view of the rule they want. - The further removed the statement is from a direct statement about the facts remembered or believed, the more likely it will be admitted. E.g., in a trial for murder, the victims statement Im afraid of Donald is more likely to be admitted than Im afraid Donald is going to kill me. Analytically, these are the same (offered as proof of her fear, but is in effect being used to prove that her husband did something which is external to her state of mind). e. Statements for purposes of medical diagnosis or treatment 803(4) High reliability most people wont lie to their doctors when they are seeking treatment.. Statements must be made for the purposes of medical diagnosis or treatment and describing medical history, or past or present symptoms, pain, or sensations, or the inception or general character of the cause or external source thereof insofar as reasonably pertinent to diagnosis or treatment. Statements made to hospital attendants, ambulance drivers, or even members of the family might be included. Unconscious victim or minor child statements made by third party (e.g. parent) to the physician are often admissible under the exception. Experts since exception applies to statements made for diagnosis, the exception contemplates those made to a physician solely for the purpose of presenting expert testimony (most juries will appreciate the self-serving nature of these statements anyway). Fault/identity not generally admissible for this purpose (not related to diagnosis or treatment). - I was struck by a car Admissible. I was struck by a car that blew the red light. The part about the red light is inadmissible since this goes to fault and is not related to diagnosis or treatment. - Blake v. State (admitting statement of young-teen rape-victim that my stepfather did this. Court holds that 1) declarants motive was consistent with purpose of promoting treatment/diagnosis; and 2). The content of the statement was reasonably relied upon by a physician in treatment and diagnosis). - This exception (the Renville test) seems to apply to cases involving young victims of sexual abuse/assault. f. Past Recollection Recorded 803(5) Past recollection refreshed Rule 612 if you just show the witness a document or an object (can be a piece of cheesecake) in order to refresh memory silently (witness cannot read a document aloud), there is no hearsay issue. You merely need to show the item to the other side before using it. PRR is used where the witness cannot recall points sufficiently to testify to them. (e.g. a witness sees a fleeing vehicle and writes down the tag number years later at trial, she cannot recall the tag, but testifies that she accurately wrote it down at the time). Trustworthiness inherent in the fact that the statement was recorded while the events were still fresh in mind. Rule requires: 1. the witness lacks present recollection of the matter; 2. the statement accurately reflects knowledge he once had; 11

3. he made or adopted the statement; and 4. he did so while the matter was fresh in his mind. v. Example doctor performed autopsy on victim, but he does 200 per year. Prosecutor lays foundation: do you recall all of the details? (no) did you take notes of the autopsy (yes). were those statements accurate and fresh when you wrote them? (yes). The autopsy report will then be read into evidence. vi. The recorded recollection is read into evidence rather than admitted because it is still hearsay; moreover, admitting it will give this evidence an advantage over the spoke evidence that a witness is in fact able to recall (document is admitted if offered by an adverse party). vii. Use of 803(5) requires that witness does not remember! (prep your witness! Tell him that, if he doesnt remember, say he doesnt remember!). viii. Two persons (Booz a note case): X tells cop the license plate number of getaway car, and cop writes it down on a deposit slip. Admissible because the agent could verify the accuracy of his transcription and the observer can testify that he related an accurate recollection of the number to the agent. (The speaker has adopted the record). Both witnesses must testify. g. Business Records 803(6) i. A very broad exception to the hearsay rule. Theory: businesses are expected to keep accurate records. ii. Also, necessity: business records are hearsay that cannot be replicated by anyone (e.g. Enron records). iii. The witness who testifies to them must be custodial simple a person who can authenticate the document as being from that particular company. iv. Four requirements: 1. Regular business records, regularly kept business is broadly defined to include records of a business, institution, profession, or occupation or calling and includes non-profits. Records must be of a type that are regularly kept. Records include memoranda, reports, records, or data compilations in any form. Computergenerated records are routinely records. Records prepared for a criminal enterprise may qualify. 2. Personal knowledge of source record must have been made by 1) a person with knowledge of the matter recorded or 2) from information transmitted by a person with such knowledge. If both the supplier of information and the recorder of that information are part of the same business, the record is admissible; thus, 803(6) contemplates the use of double-hearsay (or triple, quadruple, etc.) Pretrocelli v. Gallison medical record inadmissible where the information at issue was provided by the patient: 803(6) doesnt apply because info not relayed as part of a business routine, and declarant had no personal knowledge of the content of the statement (that a nerve had in fact been severed by a prior physician). Could have come in under 803(6) for the record and 803(4) for the statement, but wouldnt have satisfied the strategic goal of getting the statement attributed to a physician. 3. Contemporaneity information must be recorded at or near the time of the event this goes to ensure the reliability. 4. Foundation testimony the foundational witness must be acquainted with the records management system to establish that the requirements of the exception have been satisfied. The custodian or other qualified person may lay the foundation. This witness need not have personal knowledge of the particular entry (indeed need not even be employed by the business in which the record was made). v. Trustworthiness documents otherwise admissible under 803(6) may be excluded where the source of information or the method or circumstances of preparation indicate lack of trustworthiness. (e.g. a truly self-serving record obviously prepared for purposes of trial). h. Public Records 803(8) i. Theory: the great responsibility that attends the discharge of government functions in a democracy. ii. Also necessity: we dont want to subpoena public servants every time we need information gathered during the course of their duties. iii. Three parts to the rule: 1. 803(8)(A) admission of mundane records of agency setting forth the activities of the office or agency. There is an implicit firsthand knowledge requirement. 2. 803(8)(B) Matters observed per legal duty (excluding, in criminal cases, matters observed by police officers and other law enforcement personnel). 3. 803(8)(C) In civil actions (or against the government in criminal actions), factual findings resulting from an investigation made pursuant to authority granted by law 4. Unless the sources of info or other circumstances indicate lack of trustworthiness (this safety valve applies to all three preceding parts.)

12

The police report in Baker v. Elcona Homes fit the third exception officer didnt observe the accident, but he made factual findings based upon the evidence at the scene since the trial was civil, the evidence came in. The report contained a statement of one driver, but it was not h/s under 801(d)(1) (B), a prior consistent statement. Four trustworthiness factors: (1) timeliness of investigation; (2) special skill or experience of the official; (3) whether a hearing was held and the level at which conducted; and (4) possible motivational problems. iv. FL does not allow in accident reports youll have to call the officer. v. Conclusion vs. factual finding rather than parse the difference, just realize that these are on a continuum. The more things look like a baseless opinion, the more likely the safety valve will operate to prevent admission of the document. vi. 803(6) inapplicable where a document is barred in a criminal trial by 803(8)(B) or (C), it cannot come in under 803(6) as a business record. However, if you put the person who created the document on the stand, you can use it as past recollection recorded under 803(5) if the creator of the document has no memory of the incident at issue. i. The Minor Exceptions 1. Learned Treatises 803(18) must show that treatise is 1) reliable authority; and 2) either the expert relies on it in direct exam or it is called to his attention on cross exam. Treatise may be read into evidence, but not submitted as an exhibit. 2. Ancient Documents 803(16) Documents older than 20 years (trustworthiness not an issue whatever generated the instant litigation probably didnt affect the writing 20 years ago). 3. Market reports, commercial lists 803(17) VERY IMPORTANT covers a variety of data published and generally used and relied upon by the public or persons in particular occupations. Reliance and widespread circulation ensure reliability. (e.g. price lists, stock quotes, insurance mortality tables, phone books, city directory, etc.). May also include specialty magazines and periodicals. Contentious: credit reports generally admitted only to show the information available to the creditor rather than as proof of the matters contained therein. 4. Felony Convictions 803(22) Admitted in narrow circumstances to prove facts essential to sustain the judgment. Not admissible against a D in a criminal except for purposes of impeachment (FRE 609). 5. Absence of Record 803(7) and 803(10) former for business records and latter for public records. Where you want to prove the negative, you may offer proof of the absence of entries as evidence of the nonoccurrence or nonexistence of a matter that one would expect to be recorded in is such places (e.g., as proof that X doesnt exist, credit records, city directories, and various public records, none of which contain information on X). 6. Birth, Marriage, and Death 803(9) proof of milestones available in records of vital statistics. 7. Records of religious orgs. 803(11) churches and synagogues dont generally lie. May be used to prove marriage, death, blood relation, ancestry, etc. 8. Marriage, baptismal, and similar certificates 803(12) 9. Family records 803(13) as proof of personal or family history (e.g. Bibles, engravings on rings, inscriptions on portraits, engravings on urns or tombstones, etc.). 10. Real property, documents of conveyance 803(14), (15), (20). (14) permits public records of real property to be admitted, (15) allows statements contained therein to be used for the truth of what they assert. (20) allows evidence of reputation as to boundaries or of customs affecting lands in the community (e.g. where everyone agreed at one time that a particular river was once navigable). 11. Reputation evidence 803(19), (21), (23) (19) used a lot authorizes proof of reputation of a person within his family concerning matters of birth, marriage, death, and relationship by blood and marriage (e.g., we only know our birthday because someone else told us!). (21) allows proof of reputation within the community as to character. (23) allows evidence of judgments as proof of personal/family matters or boundaries, where the finding is essential to the judgment. Rule contemplates that such matters have been carefully litigated. G. Hearsay Exceptions FRE 804 Declarant Unavailable a. This rule requires necessity these exceptions represent hearsay that is good enough to be admitted, but are not as good as live testimony (without the necessity requirement for 804 exceptions, the whole live trial system would collapse). b. Unavailability Requirement 804(a)(1)-(5) Means unavailability of testimony by X, not that X is physically unavailable. Ruled on by judge under 104(a). 13

i.

ii.

i.

ii. iii. iv. v. vi. i. ii. iii. iv. v.

Five definitions provided by 804(a). 1. Claim of privilege (e.g. 5th Amendment) (technically must be called to the stand before claiming privilege). 2. Refusal to testify contemplates actual refusal, and an effort to secure cooperation, including the threat of contempt, is essential. 3. Lack of memory a declarant who testifies that he does not remember the subject matter of his prior statement. 4. Death, infirmity, or illness (must be a major illness). Just because a witness is unavailable for purposes of the Rule due to mental illness, that witness is not necessarily barred from being competent to give evidence. 5. Unavoidable absence if presence cannot be obtained by subpoena or other reasonable means. Rule contemplates the use of deposition, so you need to try and get one before invoking 804. You cannot procure absence through wrongdoing. c. Former Testimony 804(b)(1) Requires: 1) hearing or other proceeding or deposition; and 2) same party offered against now as in prior proceeding (or, in a civil case, a predecessor in interest with the similar opportunity or motive to develop the testimony). - Criminal trials are strict and have no predecessor in interest exception. - E.G: U.S. v. A,B,C, and D and trials have been severed. If a star witness X against all Ds testifies against A and is fully cross examined by As attorneys, and then X dies, Xs testimony is inadmissible against B, C, or D. - Civil trials are less strict. Once we determine that there is in fact a predecessor in interest, we then need to determine if there was a similar motive to develop testimony and cross examine (or direct) (fully developed testimony and same motive is the basis of reliability). - Lloyd v. American Export Lines. Famous for Judge Aldisert reading out the predecessor in interest requirement. The following hypo illustrates the idea behind the case (totally different facts): A v. Airline (X is a star witness on damages), then X dies. A v. Manufacturer (A wants to admit Xs testimony against M). Based on the rule, Airline is not a predecessor in interest of M; thus, even though the motive for cross exam is the same, and Xs testimony (were assuming) was fully developed, the testimony is inadmissible. However, based on Lloyd, the court would focus only on the motive and development of testimony, and would admit the testimony. Majority view (and the one Seigel thinks is right from a purely evidentiary point of view). Transcript most commonly used. Double hearsay since there is a court reporter. 805 allows double hearsay, and 803(8) takes care of the reporter and 804(b)(1) takes care of the contents assuming declarant is unavailable. Administrative hearings ok no need for judicial proceedings. You may try 801(d) for prior inconsistent statements, and if the witness says I dont remember, he is unavailable for purposes of 804 and you may try to use 804(b)(1). Grand jury testimony is not acceptable (no cross-examination). Preliminary proceedings not usually acceptable rarely if ever is the testimony fully developed. d. Dying Declarations 804(b)(2) Based on the theory that a godfearing person will not meet his maker with a lie upon his lips. Also, dying declarations are likely to speak of facts in the forefront of memory. Most often used in criminal trials. The only statements admissible under the exception are those that go to the death itself. Doom must be unequivocally impending! This is decided by the judge under 104(a). e.

Declarations against interest 804(b)(3) Although statements against interest may be admitted under the admissions doctrine of 801(d)(2), Rule 804 has several differences: 804 requires personal knowledge; the statement must be adverse or against interest; declarant must be unavailable; and statement need not have been made by a party. ii. Theory: trustworthy on the ground that a person is unlikely to state facts (or make statements) harming his own interest unless they are true. iii. Though not explicit, firsthand knowledge by the declarant is required. iv. Against interest requirement focuses on the declarants situation and motives at the time the statement was made. i.

14

v. vi.

vii.

i. ii.

iii.

Context is very important: I owe Jim $500 is a statement against declarants pecuniary interest (acknowledges a debt); however, if the statement was made in the context of a dispute over an alleged $1,000 debt, the statement is not against interest. Criminal Context a statement tending to expose the declarant to criminal liability is only admissible in two narrow circumstances: (A) if offered to exculpate an accused, it is supported by corroborating circumstances that clearly indicate its trustworthiness; or (B) if offered to inculpate an accused, it is supported y particularized guarantees of trustworthiness. Williamson holding that, in a criminal context, non-self-inculpatory statements, even if they are made within a broad narrative that is generally self-inculpatory, are inadmissible (i.e. collateral or associated statements dont come in). Court takes the narrow view of statement but does not preclude naming another party. This is consistent with Bruton because the trials may be separated. - ACN: suggests that it is ok to include collateral statements related to the precise statement against penal interest, subject to two limits (note: this is the same as Dissent in Williamson as well as the McCormick view): 1. Should exclude a self-serving collateral statement due to unreliability (e.g. may be shifting blame); and 2. Statements made under circumstances where it is likely that the Declarant had a significant motivation to obtain favorable treatment (e.g. offers for leniency) then the entire statement should be inadmissible. - Context is always important if it appears that the statement was made in a context such that it was truly not against interest (e.g. trying to shift the blame or share the blame), the statement should not come in. f. Roberts, Crawford and the Confrontation Clause Confrontation clause has two prongs: 1) necessity; and 2) reliability. Roberts Appears to allows statements against interest based upon a theory of reliability regardless of cross examination (only cross exam in Roberts was at a preliminary hearing, which is usually insufficient, e.g. for 804(b)(1)). Two ways to determine reliability: 1. Firmly rooted hearsay exceptions reliability may be inferred (includes coconspirator statements, excited utterances, statements for medical diagnosis or treatment, business records, dying declarations, agents admissions and public records Supreme Court, White v. Illinois). Effectively, the first necessity prong of the confrontation cause is satisfied when dealing with firmly rooted exceptions. 2. Non-firmly rooted hearsay exceptions require indicia of reliability (particularized guarantees of trustworthiness . . . inherent from the circumstances Idaho v. Wright). (Includes catchall exception and against-penal-interest exception. Crawford takes the latter off the table). Crawford finally settled at least one issue: 1. Where statements are testimonial, they are inadmissible against a criminal D unless the declarant is both unavailable and only where the D has had a prior opportunity to cross-examine. 2. The Confrontation Clauses goal of reliability is a procedural rather than substantive guarantee: It commands, not that evidence be reliable, but that reliability be assessed in a particular manner: by testing in the crucible of cross-examination. 3. YOU CANT DISPENSE WITH CONFRONTATION MERELY BECAUSE TESTIMONY IS RELIABLE. 4. Testimonial includes: prior testimony at a preliminary hearing, before a grand jury, or at a former trial; and statements taken by police officers in the course of interrogation. Post arrest statements will no longer come in under the against interest exception subsequent to Crawford. 5. Non-Testimonial: falls under hearsay law (i.e. not under Confrontation Clause) heres the ambiguity: does this bring back Roberts (i.e. looking for other indicia of reliability in a criminal case? Or does it automatically come in since not testimonial?) Seigel doesnt answer this. 6. United States v. Trala, 3d Cir., No. 02-4524, 10/26/04) Recent Post-Crawford case: Stopped in their car following a bank robbery, Ds girlfriend made false and conflicting statements to police officer. She doesnt testify, but her statements offered at trial to show Ds consciousness of guilt (i.e. not for their truth) Held: D has no right to cross-examination declarant where the statements are not being admitted for their substantive truth. IV. Relevance Revisited A. Character Evidence FRE 404 & 405 a. General Ideas Character Broad describes the essence of a person in a combination of qualities. 15

i.

ii. iii. iv.

v.

vi. vii. i.

ii.

i. ii.

Character narrow describes specific inclinations of a person and suggesting their innateness in him (e.g. person is by nature cautious, careless, violate, brave, etc.) In a narrow sense, character has great probative value and leads to the propensity argument. Propensity argument using proof of character as substantive evidence of conduct on a particular occasion this argument, though probative, is necessarily prejudicial, and the rules only permit limited use of character evidence. 1. Prejudice paints the D as a bad person. 2. Prejudice overvaluation of character evidence by the jury. Form of evidence three ways, all require a character witness. 1. Specific acts (e.g. evidence that D falsified a document as proof of untruthful/deceitful character). 2. Opinion (e.g. witness thinks D is a belligerent asshole as proof of violent character). 3. Reputation only form permitted at common law (obviously contemplates multiple hearsay, but notice that 803(21) provides the needed exception for this). Sneaky Character Evidence watch out for the following: 1) Mug shots; 2) Rap sheets; 3) Prior arrests; 4) Profile evidence; and 5) syndrome evidence. D opens the door Character evidence is generally inadmissible, but the D may open the door to admission: we provide the D every ability to exculpate himself, subject to the ability of the P to rebut Ds evidence. b. Proving Character 404(a) Generally, evidence of character is not admissible for the propensity inference (404 is a rule of exclusion), but may be allowed in under 404(a)(1), (2)(two reasons), or (3). 1. 404(a)(1) the defense may offer character evidence regarding the accused. If this is done, the prosecution may then offer evidence in rebuttal to counter only the particular trait raised by the D. Criminal cases only. 2. 404(a)(2) the defense may offer character evidence regarding a victim. Again, the prosecution may then offer evidence in rebuttal to counter only the particular trait raised by the D. Criminal cases only. 3. 404(a)(2) Also, if D claims that the victim was the first aggressor leading D to act in self-defense, the prosecution may present evidence of the victims peaceful character. (note: P need not wait for case-inrebuttal: Ds claim of self defense is in the pleadings and automatically triggers this part of the rule). Criminal cases only. 4. 404(a)(3) Evidence of character of a witness as provided in FRE 607, 608, and 609. Form of proof Governed by FRE 405. 1. 405(a) If character evidence is allowed in under 404, it may only be in the form of opinion or reputation. Except, on cross-exam, inquiry is allowable into relevant specific instances. 2. 405(b) Allows evidence of specific instances of conduct only where character is an essential element of a charge. (Character itself would have to be an ultimate issue in the case). (1) If specific instances of past crimes were always admissible, we would have the systemic prejudice that Seigel calls the Casa Blanca effect (round up the usual suspects). Police would have the incentive to find others previously found guilty of the same crime that they are presently investigating. (2) Criminal cases as a practical matter, never used (ACNs had to use seduction as an example). - Entrapment one defense where it is plausible to say that character is an element D essentially claims that governmental action induced an otherwise law-abiding individual to commit crimes. (3) Civil cases often an issue (e.g. defamation, negligent entrustment, child custody). - In these instances, the character evidence is offered as an end in itself rather than a mere predicate fact supporting an inference of behavior on a given occasion. c. Rebuttal vs. Cross Exam Rebuttal once the D opens the door, the P may walk through it. May only use the same methods as D (opinion and reputation); and evidence must reflect on the same trait. Cross exam P may cross examine Ds witness, but P is no longer limited to opinion and reputation Specific instances relevant to the trait raised by D may now be introduced. 1. Theory is that cross-exam tests the accuracy of the character witness testimony. 2. Must be supported by a reasonable (i.e. good faith) basis (because the question can be more damaging than the answer, and there is no 403 argument.). 3. Must go to the pertinent trait or character. 4. Extrinsic evidence of specific instances is inadmissible (we dont want a trial within a trial). 5. Remoteness acts used in cross-exam should not be too remote. 6. Re-direct case law has provided that D may rehabilitate a witness, who had been asked about specific instances, by asking about contrary specific instances. 16

iii.

i. ii. iii. iv.

i. ii.

Form of questions on specific instances: 1. Opinion evidence: Well, that is your opinion, but did you know (e.g. that D beat the crap out of his wife last year?) 2. Reputation evidence: Well, have you heard (e.g. that D beat the crap out of his wife last year?) d. Foundation and other issues Obviously, the character witness needs to be asked enough questions to provide a foundation for the testimony (e.g. for opinion, how do you know D? or for reputation, are you a member of the community? what have you heard? etc.) Timing a witness knowledge of reputation at the time of the alleged offense is relevant not the persons knowledge at the time of trial. Pertinent trait a character trait must be relevant to the crime(s) charged e.g. peaceful character in crimes of violence; honest character for theft, etc. law abiding D may introduce Ds trait of being law abiding, but this is a double edged sword while a trait of honesty may only be rebutted by dishonesty, the rebuttal of law abiding may include just about anything that is unlawful. e. Character Evidence in Civil cases 404(a) cannot use character evidence for the propensity inference in civil cases. Only 404(a)(3) applies to civil cases. B. a. b. c. 404(b) Evidence of Other crimes, wrongs, or acts 404 does not preclude the use of other acts evidence to prove matters other than character. 803(22) provides for the use of convictions (evidence of other crimes is classic hearsay). Acts need not be criminal, and need not be similar (except in cases of evidence to prove MO, where remarkable similarity is the theory of admissibility). d. 404(b) permits use of such instances to prove, e.g., motive, intent, lack of mistake, identity, MO, etc. e. Analysis: First: Rule 401 (404(b)) identify a material issue, other than character, for which the evidence is offered. Because other purposes is so often abused by prosecutors to sneak in character evidence, the prosecutor must establish a relationship between the proffered evidence and an essential element of the charged offense. Second: Rule 403 Balancing test (judge has major control here, unlike in 404(a)) if probative value is substantially outweighed by prejudice (may be unfair prejudice, confusion, or misleading), evidence is excluded. (1) The issue must be in dispute there is some circuit split on this, but, generally, if an issue is not in dispute, the probative value of the evidence declines significantly. (2) Limiting instruction basically, dont infer character, this only applies to, e.g., identity. Often, these have the opposite effect: A drop of ink cannot be removed from a glass of milk. Third: 104(b) Conditional relevance determine whether there is prima facie evidence of Ds involvement in the other act. (1) Huddleston v. U.S controlling on this issue: a preliminary finding by the court that the govt proved a prior act by a preponderance is not required! Judge makes a threshold decision whether the evidence is probative of a material issue other than character. (2) If there is evidence sufficient to support a finding, the evidence may come in. The evidence is relevant if the jury can reasonably conclude that the act occurred and that D was the actor. (3) Strength of the evidence establishing the act is a factor in the 403 test. f. Proof of identity: Preparation D steals gun and later commits murder. In the murder trial, evidence of the theft is relevant on the issue of identity and is admissible, notwithstanding the fact that the jury will likely infer Ds larcenous character. Identity through motive e.g. evidence of Ds affair with victims wife in a murder trial. MO evidence where there is a striking resemblance between two similar offenses. D may introduce reverse 404(b) evidence tending to show that another person is culpable. Because you are trying to argue that D has a given signature, it needs to be pretty darn specific to overcome the 403 objection that will likely be raised. g. Proof of Intent: E.g. D defends drug distribution/conspiracy charge by arguing lack of intent. P may offer evidence of prior acts of drug dealing, not for the propensity inference, but on the issue of intent. h. Proof of plan or design: Conspiracy a tricky little bugger: Assume a charge of conspiracy to sell drugs and four substantive drug counts. Prosecutor may successfully introduce evidence of uncharged drug offenses, not for propensity, but as proof of the ongoing agreement to continue committing the substantive offense. Here, the other purpose in 404(b) is proof of conspiracy. 17

i. ii.

iii.

i. ii. iii.

i. i. ii. iii.

iv.

Note: Some judges would consider these uncharged offenses as part of the whole (i.e. res gestae); thus 404 is not even implicated. i. Other purposes: i. Absence of accident: in child abuse case, mother claims accident. Evidence of prior trips to the hospital during which child is treated for bruises. ii. Lots of potential prejudice, particularly overvaluation. Here, # of trips in short period of time would cut in favor of admission over the 403 objection. C. Character in Sex Offense Cases a. FRE 412 Rape Shield 412(a) applies to both criminal and civil cases. Inadmissible: 1) evidence offered to prove that any alleged victim engaged in other sexual behavior; and 2) evidence offered to prove any alleged victims sexual pre-disposition. b. Exceptions in Criminal Cases 1. 412(b)(1)(A) origin of semen, pregnancy, or disease i. Opposite 405 only specific instances are permissible may be used by accused to exculpate himself, but prosecution must make the 1st assertion. ii. Evidence not automatically admissible (e.g. girl has semen from three guys in her, but D is one of them D cannot offer proof that she slept with the other two guys all it does now is go to slutty character). 2. 412(b)(1)(B) past sexual activity with accused i. Generally limited to the issue of consent. ii. Probative value not much higher than with third parties, but jury must know the relationship between accused and alleged victim. iii. Evidence not automatically admissible, but prior sex with the victim usually gets in. 3. 413(b)(1)(C) Constitutionally required: generally implicating the 6th Amendment CC. i. Example: guy at party is told that chick is easy, go for it. He has sex and is accused of rape. Tells attorney that he wouldnt have had sex with her but for what he was told, and that her resistance seemed minimal given the information he had. This evidence goes to mens rea, and keeping it out may violate Ds constitutional right to a fair trial. c. Exceptions in Civil Cases 412(b)(2) 1. Evidence (specific instances) to prove sexual behavior or predisposition is admissible if it is relevant, not hearsay, and passes a reverse 403 test. 2. Reputation evidence is precluded unless placed in issue by the alleged victim. d. Prior Offenses by Ds in Sex Crime Trials 413-415 rules preempt 404 and 405. 413 Governs sexual assault cases 414 413 applied to child molestation cases (14 years of age is the cap) 415 413 and 414 applied to civil cases. General contents of 413-415 (1) Evidence of a Ds commission of another offense or offenses of sexual assault is admissible and may be considered for its bearing on any matter to which it is relevant. (2) Evidence of prior uncharged acts must be supported only by a 104(b) standard. (3) The propensity inference is included in any matter. (4) Convictions are not necessary, and remoteness may be 15+ years. (5) 403 applies but is slanted in favor of admission. Rationale: External get rape convictions up. Internal rape is hard to prove, and we need to tweak the rules to get more evidence in on theses cases. Argument that 412 and 413 are consistent sex in and of itself is not deviant, and the fact that one has sex is not probative (412); however, the fact that a person has committed deviant sexual acts is probative (413). D. Habit and Routine Practice Rule 406 used to prove conduct a. Evidence of a habit of a person or organization is relevant to prove conduct in conformity therewith on a particular occasion. b. Need not be corroborated, and presence of an eyewitness is not required this only goes to the weight of evidence and not admissibility. c. What is habit ACNs cite McCormick: A regular response to a repeated specific situation - habit is semiautomatic. d. Not character evidence Much better than character: 1) more likely to have occurred; thus more probative; and 2) habit does not smack of moral overtone (thus less prejudicial). 18

i. ii.

i. ii. iii. iv.

v. vi.

e.

Relevant factors: 1. Specificity (e.g. being a careful driver is character and not admissible, but evidence that X stopped at a given intersection each day is habit due to its specificity. 2. Repetition if X stopped at the intersection twice a day, that is more probative than once per month (argument how many times does one have to do something for it to be habitual?) 3. Duration has X stopped at that intersection every day for five years? More probative than doing so for two-months. 4. Semi-automatic nature the more non-volitional the better (e.g. using right foot to brake a car. But, observing the Sabbath (e.g. as an alibi) is inadmissible as habit due to its volitional nature). However, some nonuniformity may be accepted (e.g. forgetting to brush your teeth one morning does not mean it is no longer a habit.) f. Routine business practices standard operating procedures are generally admissible. Best to have a person with personal knowledge of those practices testify. g. Custom to establish standard of care though custom/habit may be used to establish conduct in conformity therewith, use for other purposes is not covered by the rule. h. Rule 403 application 406 provides that habit is relevant judge has final say under 104(a), and 403 applies. E. Subsequent Remedial Measures FRE 407 a. After an alleged harm or injury, remedial measure are generally inadmissible to prove negligence, culpable conduct, a defect in a product, a defect in a products design, or a need for a warning or instruction. b. May be used for another purpose such as proving ownership, control, or feasibility of precautionary measures, if controverted, or impeachment. c. Controverted ACNs require an automatic exclusion unless a genuine issue be present and allows the opposing party to lay the groundwork for exclusion by making an admission. A D in a produce liability suit will often stipulate to feasibility in order to preclude introduction of subsequent remedial measures. d. 403 still applies. e. Tuer v. McDonald per procedure, decedent is taken off of an anti-coagulant, provided to stabilize angina, prior to surgery. Surgery is delayed, and decedent dies from severe angina attack, notwithstanding 7 hours of surgery. Protocol is changed. Issue, is the change admissible for either 1) feasibility; or 2) impeachment? Narrow feasibility Subsequent measure only allowed when D has contended that measures ere not physically, technologically, or economically possible under the circumstances. Broad feasibility includes motives and explanations for not having adopted measure earlier (e.g. medical expertise here, a change to the initial protocol was thought to be feasible, but not advisable). Impeachment some overlap with feasibility almost anything that D says can be contradicted (i.e. impeached); thus there needs to be more than mere contradiction. Court takes a narrow view of both feasibility and impeachment and excludes the evidence. f. Flaminio v. Honda guy crashes on motorcycle evidence of subsequent remedial measure excluded. 407 applies only to measures subsequent to an accident if an accident occurs following remedial measures (e.g. a motorcycle that had not been fixed), the evidence may come in. Recalls are issued to protect themselves (of course, manufacturers may still do a cost benefit analysis).

i. ii. iii. iv. i. ii.

F. Settlement Negotiations Civil Settlements FRE 408 a. Evidence of settlement negotiations and offers to compromise is generally inadmissible to prove liability for or invalidity of the claim or its amount. b. Also excluded: statements made in compromise negotiations as well as the offer or completed compromise itself. c. Theory: 1) Relevance (internal reason) the compromise may be motivated by a desire for peace rather than from any concession of weakness of position (danger of overvaluation); therefore the evidence is often irrelevant; and 2) public policy (external reason) we want to encourage compromise and settlement of disputes. d. Evidence discovered during negotiations, that is otherwise discoverable, is not precluded merely because it was discovered during negotiations. e. Claim must be disputed as to either validity or amount. (e.g. not implicated if the effort was to induce a creditor to settle an admittedly due amount for a lesser sum: neither the claim nor the amount is in dispute) i. Sales rep for herbicide tells farmer, whose crop has obviously suffered due to the use of the product, dont worry, Well take care of you. Later, rep tells farmer, You just tell us the damages youre claiming, and well bill the company. ii. The first statement is clearly admissible as an admission under 801(d)(2)(D) and is not barred by 408 there is no dispute. iii. The second statement, although it utilizes the word claim, is still not barred by 408 again, no dispute, f. Evidence may be offered for another purpose (e.g. proving bias or prejudice of a witness). 401 and 403 apply. 19

i.

E.g. car accident between A and B. Passenger C (in Bs car) settles with A, but A defends against Bs claim. B cannot introduce evidence of negotiations between A and C for the purpose of proving liability. However, if A calls C as a witness against B, B may introduce evidence of the settlement for purpose of proving Cs potential bias.

G. Settlement Negotiations Criminal Pleas, Plea Discussions, and Related statements FRE 410 a. Evidence of 1) a plea of guilty later withdrawn; 2) a plea of nolo; 3) any statement made in the course of any Rule 11 FRCrimP proceeding; or 4) any statement made in the course of plea discussions with an attorney for the prosecuting authority which dont result in a guilty plea or result in a plea later withdrawn is inadmissible. b. Applies to both Criminal and Civil proceedings. c. If a prosecutor is not present: 410 protection is not automatically lost. The court will look to the subjective intent of the D did the D reasonably believe that he was involved in plea negotiations? d. Exceptions: i. Rule of Completeness if a statement made in the course of plea discussions has been introduced, other statements from those discussions may be introduced if in fairness they should be considered contemporaneously with it. ii. In a criminal proceeding for perjury or false statement, if the statement was 1) made by the D, 2) under oath, 3) on the record, and 4) in the presence of counsel. e. Rule is implicated early! If a prosecutor calls and says, hey, can we talk about a plea (or something similar), this is inadmissible under 410. Even at early stages, statements are damaging because there is some admission of guilty made by Ds agent. f. Running off at the mouth post-arrest will almost always come in as an admission! Telling the authorities to let your wife go since she had nothing to do with it, followed by them letting her go, will not usually be considered plea negotiations. H. Proof of Payment of Medical Expenses, FRE 409 & Proof of Insurance Coverage, FRE 411 a. 409 known as the good samaritan rule precludes admission of evidence that a party paid or offered to pay medical expenses of another party if such evidence is offered to prove liability for the injury. Theory: dont want to discourage people from aiding others. May be offered for another purpose, but 403 continues to apply. Rule doesnt exclude statements that may accompany the payment of medical services these statements are ancillary and are not necessary for the payment or offer of payment. b. 411 Evidence that a person 1) was or 2) was not insured against liability is not admissible upon the issue of negligence. Policy such knowledge leads juries to decide cases on improper grounds. Criminal cases this policy doesnt apply in a criminal case (e.g. arson) in which insurance proceeds are the motive for the crime. Other purposes proof of coverage may be offered to prove, e.g., agency (carrying coverage on another person may prove an agency relationship), ownership or control (coverage on property is probative here), or bias (e.g. an adjustor as a witness may be impeached as representing a company with a financial interest in the litigation). 403 applies. V. Witness Competency a. FRE 601 Every person is competent to be a witness except as otherwise provided in these rules. Competency is ultimately ruled on by the judge under 104(a). U.S. v. Lightly a criminally insane inmate may still testify unless it can be shown that the witness does not have personal knowledge, that he lacks the capacity to recall, or that he does not understand the duty to testify truthfully. There are no mental or moral qualifications for testifying as a witness, and discretion is exercised in favor of allowing testimony. Interest in the outcome and mental capacity are relevant to witness credibility, but do not bar testimony (major change from old common law). ACN a witness wholly without capacity is difficult to imagine. The question is one particularly suited to the jury as one of weight and credibility. b. FRE 602 Personal Knowledge Requirement A witness may not testify to a matter unless evidence is introduced sufficient to support a finding (i.e. 104(b)) that the witness has personal knowledge of the matter. Evidence of personal knowledge may come from the witness or other sources. Hearsay note: a witness may testify to a hearsay statement, if otherwise allowed by the hearsay rules, if he has personal knowledge of the making of the statement; however, 602 would prevent the witness from testifying to the subject matter of the hearsay statement if the witness has no personal knowledge of it. 20

i. ii. iii.

i. ii. iii.

i. ii. iii. iv. v. i. ii. iii.

iv.

i. ii. iii. iv. i. ii. iii. iv. i. ii. i. ii. i. ii. iii. i. ii. iii. iv.

i.

ii. iii. iv. v.

In sum, if the source of information is hearsay, there is no personal knowledge; but if the source is not hearsay, then there is personal knowledge. (e.g. IRS examiner testifies to fraudulent tax returns based upon an examination of the returns and speaking to the taxpayers source is hearsay, but perhaps gets in as statement against interest). c. FRE 603 Oath Requirement No special verbal formula is required. Affirmation is simply a solemn undertaking to tell the truth. U.S. v. Fowler Tax protestor denied ability to testify for refusal even to recite I state that I will tell the truth in my testimony. Conviction upheld on appeal. Oath is required for any subsequent perjury prosecution. Judge concluded that D refused to awaken his conscience and impress [his] mind with the duty to testify truthfully. Lack of oath = inadmissible testimony. Failure to object to lack of oath leads to waiver. d. Children as Witnesses Main concern is whether child knows difference between truth and falsity. Child has to understand 1) the difference between fantasy (untruth) and reality (truth); and 2) needs to say in his/her own words that he/she will tell the truth (603). Confrontation Supreme Court held that you cant put a screen between the witness and D. Closed circuit TV is permissible on a case-by-case basis if needed to protect the child, but not as a matter of course. Excited utterances are often used if a child cannot or does not testify: he tried to rape me (yelled right after event) will be admissible. No Crawford issue because statement is not testimonial. e. Dead Mans Statutes Limit in differing ways testimony about transactions with deceased persons. Designed to protect estates against fraudulent claims such statutes are highly criticized. f. Lawyers as Witnesses Lawyers may testify subject to the MRs. MR 3.7 generally prohibits the undertaking of representation if the lawyer is likely to be called as a witness. A-C privilege still applies. g. Judges as Witnesses FRE 605 Broad prohibition against judge testifying in the same trial over which he/she sits as judge. No objection is necessary to preserve this. A judge may be called in another trial or hearing, or in a post-trial proceeding in the same case. h. Jurors As Witnesses FRE 606(a) 606(a) provides that a member of the jury may not testify as a witness before that jury in the trial of the case in which the juror is sitting. Theory: how cant a juror remain impartial after testifying for one of the parties? A party must object to a juror being called, but the rule requires that the object may be made away from the jury. 606(a) does not bar the judge from interviewing the jury if there is evidence of jury malfeasance (e.g. reading a newspaper account of the instant trial), the judge may interview the juror(s) on the matter, usually in front of the parties counsel. i. Postverdict Testimony by Jurors FRE 606(b) Jurors are incompetent to testify about the validity of a verdict or an indictment if the subject of their testimony involves internal influences. This bar includes affidavits. Internal influences include: 1. The method or arguments of the jurys deliberations; 2. The effect of any particular thing upon an outcome in the deliberations; 3. The mind set or emotions of any juror during deliberations; and 4. The testifying jurors own mental process during the deliberations. Rule only precludes post-verdict inquiry prior t the verdict, the judge may inquire as he/she so desires. External influences may testify to extraneous prejudicial information that has been introduced into the jury deliberation process (may include newspapers, or a bailiffs statements). Protections against an impaired jury: 1) Voir dire; 2) jury is observable by the court; 3) jurors are observable by each other; and 4) a party may seek to impeach the verdict by nonjuror evidence of misconduct (if someone else has personal knowledge of juror misconduct, that person may testify). Examples: 1. $800m error attack verdict directly through JNOV or remittur, depending on law. (misinterpretation of judges instructions is a subject upon which jury is incompetent to testify). 2. Jury goes on personal fact finding mission whatever was found is admissible wholly external. 3. Juror has almost expert knowledge on an aspect of the case (e.g. bomb-making) should have been uncovered in voir dire, and is now an internal influence. Argument that juror is now acting as an unsworn expert usually loses. j. Religious Beliefs or Opinions FRE 610 21

i. ii. VI. A. a. b. c.

Inadmissible to show that by reason of their nature, the witness is either more or less credible. An inquiry for the purpose of showing interest or bias because of these beliefs is not within the prohibition.

Examination of Witnesses Direct Examination Comprised of nonleading questions FRE 611(c). A leading question suggests the answer it is much more effective to get an answer without using a leading question. Because direct exam usually involves a friendly witness, there is a great ability to lead. Other side may not object since a series of leading questions may make the testimony look extremely weak. d. Exceptions witness is 1) young, 2) timid, reticent, reluctant, or frightened, 3) ignorant, uncomprehending, or unresponsive; or 4) infirm. Also: i. Hostile witnesses may ask leading questions very little danger that attorney can put words in mouth ii. Preliminary matters non-leading qs are a waste of time. iii. Undisputed matters iv. Experts little chance of getting an expert to cave into pressure from counsel. v. Refreshing memory May refresh with leading questions, e.g. trying to put a witness at a meeting with D. Who was there? witness names people. Was anyone else there? still no answer with the D. was the D there? (youll probably draw an objection), even if sustained, youve certainly refreshed the Ws memory! Now, was anyone else there? . . . . oh yea, the D was there too! (mission accomplished). B. Cross Examination a. Leading questions are ordinarily permitted. 611(c) b. Leading questions may be disallowed when the cross-exam is more form than substance (e.g. cross exam of your own client after he/she is called by the other party). c. Scope cross exam is limited to the subject matter of the direct exam and matters affecting the credibility of the witness. 611(b). (a series of cross, re-direct, re-cross, re-re-direct, re-re-cross, etc. will seriously narrow scope). d. You may ask about witness preparation 612 This is a MAJOR line of impeachment. e. Most impeachment occurs on cross intrinsic impeachment: impeaching the target directly. C. Writing used to Refresh Memory FRE 612 a. A writing may be used to refresh memory for the purpose of testifying 1) while testifying; or 2) before testifying. i. If while testifying, the opposing party may inspect and introduce the writing into evidence. ii. If before testifying, and the court in its discretion determines it is necessary in the interests of justice, the opposing party may inspect and introduce the writing (this is a major departure from common law). - Julian v. Raytheon led to this change in common law. Court will conduct a balancing test to determine whether allegedly privileged documents used to prepare witnesses need to be turned over. - The judge will effectively determine whether the privilege has been waived. - Your client will be asked about preparation! Play by the rules and tell your client that youre playing by the rules. Some clients will assume that youre being sneaky by preparing witnesses. iii. for the purpose of testifying included to proscribe use of Rule as a pretext for wholesale exploration of an opposing parties files. D. Excluding Witnesses a. FRE 615 Witness Sequestration at the request of a party the court shall order witnesses excluded so that they cannot hear the testimony of other witnesses, and it may make the order of its own motion. Long standing rule in common law to discourage and expose fabrication, inaccuracy, and collusion (avoid pollution of future testimony). Rule is as of right, and there is no time cutoff for requesting that witnesses be excluded. Limits on scope: 1) doesnt authorize exclusion during opening statements (3d cir.); 2) may not direct witnesses not to confer privately (10th Cir). b. Exclusions from the Rule A party who is a natural person. An officer or employee of a party, which is not a natural person, designated as its representative by its attorney; A person whose presence is essential to the presentation of the partys cause (e.g. an agent who handled the litigated transaction or an expert needed to advise counsel; in criminal cases, investigative agents, generally). A person authorized by statute to be present. 22

i. ii. iii. iv. i. ii. iii. iv.

c.

Violations generally does not prohibit the witness from testifying, but subject to contempt and testimony is open to comment to the jury by reason of his conduct.

VII. Impeachment of Witnesses A. Methods of Impeachment, Overview a. Definite but nonspecific 1. Show that witness has some bias, animus, motivation, or corruption that may lead to fabricated or shaded testimony. (no rule) 2. Show a defect in sensory or mental capacity (perception or memory). (no rule) 3. Showing that witness is untruthful by nature. May do so by: i. Cross exam re: non-conviction misconduct casting doubt on honesty. 608(b) collateral, cannot be done extrinsically. ii. Cross exam re: convictions for certain kinds of criminal acts. 609. iii. Testimony by a character witness that the target witness is untruthful. 608(a). b. Specific but indefinite 1. Showing that the witness has made a prior inconsistent statement; 2. Contradicting the witness showing that he is just plain wrong on one point or another. c. Note: You may impeach your own witness FRE 607. B. Nonspecific Impeachment a. Bias and Motivation No rule on point. Credibility of a witness is always relevant. Examination for bias comes in under 401, 402, balanced by 403. BIPC bias, interest, prejudice, and corruption never collateral! May be proven extrinsically. (e.g., you ask, werent you paid $1m to lie in this trial? witness denies this. You may call other witnesses on this issue youre not stuck with the answer). Court exercises little control over BIPC, imposing only reasonable limits and cutting off testimony when the point has been made. U.S. v. Abel witness X denies being a member of a liars club in prison. Witness Y allowed back to stand to testify that witness X was a member because Y was a member with him and Abel this all goes to show that members of the same club tend to have a bias in favor of one another, which is why the evidence is relevant to show bias and may be shown collaterally. (If Y and Abel were not a members of the club with X, X could be asked directly about his membership under 608(b), but youd be stuck with his now answer non-conviction misconduct cannot be proven extrinsically). Experts questioning about how much theyre getting paid is permitted to show bias. There may be a cutoff on questions tending to show the experts total income (maybe getting too personal, and judge will invoke 611 to avoid harassment). b. Sensory, perception, communication, and mental capacity. Never collateral extrinsic evidence may always come in. Usually attacked on cross-exam first. Questions regarding drug/alcohol abuse, mental afflictions, and illness is proper. Guided only by 401, 403, and 611. c. Character for Truth and Veracity 608, 609. 404(a)(3) provides that evidence of untruthful character can be admitted to prove that a witness may be lying in court. 608 and 609 provide the roadmap for this form of attack. Only allowed to prove character for truthfulness cannot come in for other purposes. Rule 608(a) permits opinion and reputation evidence of a witness character for truthfulness or untruthfulness, but evidence of truthful character is admissible only after attack for untruthfulness. - As a practical matter, 609(a) is rarely used. 404 is used much more frequently. Rule 608(b) Specific Instances of conduct 1. Always Collateral! May only be raised on cross-examination of the target witness (counsel must have a goodfaith basis for the question since the Q itself is likely to be very damaging). 2. Because collateral, you are stuck with the witness answer. If witness is contradicting a former statement in a deposition, you may refresh memory by showing the document, but you cannot admit the document (extrinsic). 3. May only ask about specific instances or uncharged crimes that bear and truthfulness arson, burglary, murder, and others of that ilk are not relevant on this point. 4. Note that a government witness may be questioned about crimes on the basis if bias since the witness likely cut a deal with the prosecution in order to turn states evidence.

i. ii. iii. iv.

v.

i. ii. iii. i. ii. iii. iv.

23

v.

608(b) evidence is generally disfavored the rule begins by saying that you generally cant admit it; and it provides that courts discretion should be utilized before the evidence comes in basically 403 slanted slightly toward exclusion. Rule 609 Prior Convictions NOT COLLATERAL 1. When required, extrinsic evidence is usually a certified copy of the conviction. 2. Applies in both civil and criminal cases, and applies to all witnesses, including the D. 3. No contest pleas are admissible, but arrests and indictments are not (the latter may come in under 608(b) if it reflects untruthful character). 4. 609(a)(1) does not require that the felony relate to truthfulness: 403 or reverse 403 is applied. 5. Witness other than D subject to 403, and conviction must be a felony. 6. D as witness reverse 403 (though substantially not in rule) must be a felony. Gordon factors determine admissibility: (1) nature of crime (i.e. a crime reflecting on veracity has high probative value). (2) Age of prior conviction (remoteness one year old crime more probative than 8 year old crime). (3) Similarity of crime (more similar = more prejudicial: big danger of propensity inference) (4) Need for Ds testimony (if only D can provide evidence, argument for exclusion is stronger). (5) Centrality of credibility at trial if a major issue (e.g. he said, she said), such evidence is more important. (6) Other factors e.g. a long rap-sheet cuts in favor of admission also, circumstances of prior offense. 7. Lipscomb Court may inquire into the background facts and circumstances of the crime when deciding upon admissibility must be done outside of the jurys hearing. 8. 609(a)(2) Crimes of dishonesty always come in, regardless of punishment, and regardless of who the witness is (fraud, forgery, perjury, filing a false police report, and other crimes involving deceit). 9. Methods of proof generally, nature of the crime, time of conviction, and punishment: aggravating circumstances are not admissible. 10. Other purposes if evidence is coming in for another reason, e.g., bias, 609 does not apply. 11. 609(b) Ten year limit: later of 1) conviction or 2) release from confinement. Exception if evidence is extremely probative (bias toward exclusion). Measure back from date of testimony. 12. 609(c) Pardon and annulment if based on finding of innocence, evidence inadmissible. If based on rehab, inadmissible unless witness has committed another qualifying crime since the pardon. 13. 609(d) Juvenile adjudicates are generally inadmissible, unless offered for some other purpose besides character impeachment (e.g. bias). 14. 609(e) pendency of an appeal is admissible, but the pending appeal doesnt render the conviction inadmissible. d. 608(b) vs. 609 Counsel may not elect the former where there has been a conviction. This may be attractive if it were permitted, since 608(b) allows more information about the prior act than does 609.

5.

C. Specific Impeachment Prior inconsistent statements and Contradiction a. Prior Inconsistent Statements FRE 613 i. Any prior statement, whether oral, taped, written, sworn, or unsworn may be used to impeach (inconsistent conduct does not fall under this or any other FRE). ii. Must be the witness own statement. iii. This rule does not relate to 801(d)(1)(A) admissions by party-opponent. iv. Inconsistent may be directly inconsistent, a prior omission of a material fact, a prior lack of knowledge on the matter the W is now testifying about, or, in some circumstances, a present lack of memory. v. Surprise! You need not show the statement to the W; thus, you may surprise the shit out of him! However, must show statement to opposing counsel upon request. vi. 613(b) Explanation by W witness must be able to provide an explanation for the prior statement at some time during the trial. vii. No foundation requirement prior to using 613 preserves element of surprise. As a practical matter, some foundation must be laid so that the testimony is not confusing. viii. 613(b) Extrinsic evidence permissible IF witness is able to explain and opposite party may cross examine, or as justice otherwise requires (if you purposely sandbag, however, and the W is no longer available, youre not going to win the justice otherwise requires argument). ix. Subtle difference 613(a) relates to all statements whereas 613(b) relates only to prior inconsistent statements. b. Contradiction the final line of impeachment. i. Simple use: one witness may be used to contradict another witness (e.g. on the safety of a medical procedure).

24

ii.

Extrinsic evidence of contradiction (also for 613(b)) is only admissible if the matter is not collateral; thus, calling W2 to contradict W1 is only permitted if the matter is non-collateral. iii. Contradiction is not collateral if 1) otherwise relevant (i.e. relevance other than the impeaching effect); or 2) otherwise offered for another purpose (e.g. bias). iv. Two Examples: 1. Husband says that wife wore a blue dress. Calling wife to say that she was actually wearing a green dress is not permitted because the issue is collateral (i.e. a trivial matter no relevance other than the impeaching effect). 2. However, if a rape case that husband says he witnessed, calling the wife to contradict the Husband on the dress color is no longer collateral: one would think that the H would remember such a detail, and his inability to do so suggests that he may be lying. v. Collateral stays collateral if evidence falls under 608(b) and contradiction, you are stuck with intrinsic evidence only. A matter deemed collateral (all 608(b)), remains collateral. (Note: try to argue bias, which is never collateral). D. Repairing Credibility a. Two limits: 1. 608(a) directly states that repair must come following an attack on truthfulness; and 2. repair should be made at the point of attack. b. There are four common pre-emptive methods that may be used when impeachment of your witness is almost certain: 1. adduce testimony by expert regarding remuneration. 2. Prosecution or defense may bring out a witness past convictions. 3. Prosecutor may bring out a plea bargain of his own witness; 4. Calling party may bring out any connection or affinity that she has with the witness (obvious grounds of bias). c. Evidence of good character: 1. 608(a) permits opinion or reputation testimony supporting credibility after character for truthfulness has been attacked. 2. As with all character evidence, a foundation should be laid. 3. On cross counsel may raise specific instances to impeach once again. Must have a reasonable basis. This is why 608(a) is rarely used you actually give the other side the opportunity to ask character witnesses the same questions asked to the target under 608(b), but now youre nearly getting around the no extrinsic evidence rule. 4. Medical Therapy Witness for state is asked about prior convictions and acts on direct in anticipation of impeachment by defense. Defense crosses only within scope of direct. On appeal, D contended that character had not been attacked on cross, thus character witness impermissible. Court held that, though a close call, trial judge was within his discretion to permit state to offer evidence of good character (through other ws) to repair. - Merely taking the sting out of what will likely come up on cross does not open the 608(a) door (that would be silly). - On cross, examination for bias alone, is not enough to open the 608(a) door we need an attack on veracity and truthfulness then the door is open. 5. Experts are often considered incompetent to testify as to whether a person is truthful. d. Prior consistent statements: 1. If rehab is just for something like memory, you can bring it in just on the basis of 401 relevance. 2. However, when the attack alleges false testimony or some sort of fabrication 801(d)(1)(B) is now implicated (thanks to Tome, in which the Supreme Court screws up). 3. 801(d)(1)(B) permits such statements to rebut an express or implied charge against the declarant of recent fabrication or improper influence or motive. Declarant must testify at the trial or hearing and must be subject to cross-examination. 4. Only statements that pre-date the alleged time of fabrication (i.e. before W was gotten to) are permitted (Temporal requirement). 5. Tome Supreme Court confused relevance and hearsay, and held that 801(d)(1)(B) needs to be met whether the statement is being offered for substance or for rehabilitation. VIII. Opinion and Expert Testimony A. Lay Opinion Testimony FRE 701 a. Rules contemplate that facts and opinions are on a continuum, and they differ in degree rather than kind. Both are admissible within limits this is a change from common law that disallowed opinion by lay witnesses. b. Opinion testimony by lay witnesses, under 701, must be: 25

1. Rationally based on first hand knowledge; (this is not duplicative of FRE 602 the rules are distinct major requirement for lay witness) 2. Helpful to an understanding of the testimony (helpfulness standard); 3. Not based upon expert-type specialized knowledge covered by FRE 702. c. Helpfulness standard where the facts are sufficient for the jury to draw its own opinion, the testimony is not helpful (e.g., no need to opine on whether D was speeding if radar gun evidence indicates 90mph). d. Collective Facts Doctrine certain ideas well within common experience can best be expressed, and reliably so, by means of a shorthand word or phrase. e. Testimony acceptable on: identity, quantity, handwriting, value, weight, measure, time, distance, velocity, form, size, age, strength, heat, cold, sickness, health also mental and moral aspects of humanity: temper, anger, fear, excitement, intoxication, veracity, etc. f. 701 is a rule of preference opinion testimony is admissible, but direct factual evidence is preferable. E.g. The D acted in a bizarre manner and spoke incoherently is less preferable to the D was running around naked and screaming that he was the Governor of Arkansas. g. Use of words with specific legal meaning, such as rape or insanity should be avoided, or at least explained. h. Seigel calls lay opinion restricted, but permitted. i. Law lay witness is not qualified to state what the law is. j. Look out for: 1) witness says, I guess x, y, and z (speculation) 2) watch for lack of foundation (I smelled pot how does witness know) 2) terms of art (car was totaled witness not able to assess this) B. Expert Testimony FRE 702 (who is an expert, and who can testify) a. Rule provides that if scientific, technical, or other specialized knowledge will help the factfinder, an expert may be used. Two requirements: judge must decide 1) expert qualified; and 2) that testimony will help the jury. b. Expert may be qualified by 1) knowledge; 2) skill; 3) experience; 4) training; or 5) education. c. Testimony may be in form of opinion or otherwise, but must be: 1. based upon sufficient facts or data; 2. product of reliable principles and methods; and 3. witness must have applied the principles and methods reliable to the facts of the case. d. 104(a) is used to determine admissibility. Proponent must show by a preponderance that the expert is reliable, not that he or she is correct (rule contemplates experts on either side, who may reach opposite conclusions). Testimony of the expert must rely on a sufficient basis of information. e. The Daubert factors are helpful in determining whether testimony is admissible (supercedes the old Frye test): 1. whether experts technique or theory can be or has been tested (i.e. can theory be challenged in an objective sense, or is it merely opinion or otherwise conclusory?) 2. whether the technique or theory ahs been subject of peer review and publication; 3. the known or potential rate of error of the technique or theory when applied (this can be complicated how much error is acceptable? Obviously, a test that gives a bunch of false positives will not likely work); 4. the existence and maintenance of standards and controls; and 5. whether the technique has been accepted in the scientific community (incorporation of Frye). Frye note: FL still uses this test, not Daubert thus, an astrologist can be an expert and given an opinion as long as it is accepted by others in that field. f. Kumho case established that the Daubert principles apply to all experts, not simply to those whose testimony is based on scientific knowledge. 1. Syndrome evidence e.g. rape trauma syndrome: a pattern of accusation, retraction, and then accusation (involves the cycle of guilt). Whether experts could testify to this was a question after Daubert, but Kumho and the new rule says yes however, some courts dont permit it under the test below. 2. It is the judges job to scrutinize the expert it may be a common-sense test. This is what happened in Kumho (experts major premise on whether tire underinflated or defective was ok, but minor premise really stretched the test to say not underinflated: judge says that this is bullshit, and didnt allow expert to testify; thus, case lost on DV for lack of evidence on causation). Supreme Court said that this is what the judge is supposed to do. g. Other factors used by courts both before and after Daubert: 1. Independent research? Or, was it prepared just for trial? 2. Has expert unjustifiably extrapolated from an accepted premise to an unfounded conclusion. See Joiner (noting that in some cases a trial court may conclude that there is simply too great an analytical gap between the data and the opinion proffered.) 3. Whether expert has accounted for obvious alternative explanations.

26

Whether expert is being as careful has he would be in his regular professional work outside his paid litigation consulting. 5. Whether field of expertise is known to reach reliable results for the type of opinion the expert would give (e.g., expert testimony in astrology is not likely grounded in an otherwise reliable field of expertise). h. Rejection of expert testimony is the exception, not the rule vigorous cross-examination, presentation of contrary evidence, and careful instruction on the burden of proof are the traditional and appropriate means of attacking shaky but admissible evidence. Daubert (emphasis added). i. Getting the Expert in: i. You tender Dr. X an expert in Y. Must be something 1) connected to experts knowledge; and 2) relevant to the case. ii. If you Tender Dr. X as an expert in obstetrics, youll draw an objection if you ask about pediatrics. Try not to pigeonhole your expert: perhaps tender Dr. X as an expert in medicine. iii. Opponent may conduct a voir dire even during (actually just before) direct examination. C. Expert Testimony bases of testimony, FRE 703 a. Think of expert witnesses of having two components: 1. Major premise experts general hypothesis within his area of expertise. May be applicable to many cases (e.g., embolism may be caused by 1) air in heart, 2) air in tube; 3) tube came undone). 2. Minor premise applies the general hypothesis to the facts of the case (e.g. I think tube came out). b. Experts may base their opinions/conclusions on the facts or data in the particular case, which may be those made known to the expert 1) at the hearing (a) what expert hears (unique to expert); and b) information conveyed in hypothetical questions summing up evidence previously admitted); 2) by first-hand observation before the hearing (like 602); or 3) of a sort that amount to outside data information he gleans before trial by consulting other sources. (note: personal knowledge rule for lay witnesses is severely relaxed for experts!). c. For category #3, must be of a type reasonably relied upon by experts in the particular field. Consequently, experts may rely on otherwise inadmissible evidence. 1. For hypo in the book, the medical chart may have been filled in by a person who was not even there; thus, although charts usually relied upon by doctors, this isnt the type of chart a doctor would rely upon. 2. Counter argument is that doctors rely upon charts all the time without inquiry into who wrote it. 3. Judge will balance, and decide whether the opinion comes in. (note this chart is what allowed the doctor to form her minor premise). d. Evidence relied upon need not be admitted (i.e. may be admissible but not admitted). e. Inadmissible evidence, serving as part of the basis of the experts opinion, shall not become suddenly admissible under the rule (expert should not disclose such evidence directly or through inference). You cannot get around other rules by including it in an expert opinion. You can give an opinion based upon inadmissible evidence, but you cannot tell the jury the basis for the opinion if the basis is otherwise inadmissible. f. Exception reverse 403 test for otherwise inadmissible evidence. 1. For major premise (how expert got her knowledge) usually comes in: training, experience, etc. Usually the probative value clearly outweighs the prejudice. 2. For minor premise things get more difficult: how did the expert get her information about this case. The chart in the doctor hypo, if not otherwise admissible (e.g. as a business record), could only come in if the probative value substantially outweighs the prejudice. D. Expert Testimony FRE 704 Ultimate issue restriction abolished a. Rule provides that an experts opinion or inference that is otherwise admissible is not objectionable because it embraces an ultimate issue to be decided by the trier of fact. b. Exception subsection (b) (John Hinckly exception) An expert may not opine as to whether the D had the requisite mens rea for the charged offense or defense thereto (e.g., an expert may not state that the D was in fact insane.). exam well only worry about insanity. (Seigel would have to look to see if rule applies to other mens rea). E. Expert Testimony FRE 705 (controls how testimony is presented) a. an expert may in fact give an opinion before giving a foundation. This is the opposite from other witnesses. With an expert, you basically skip to the punchline (e.g., Dr. X, is it your opinion that Lynsey has an abnormal attachment to her dog?) b. Must disclose the basis of the opinion on cross examination if probed by opposing counsel. c. The protection in 703 is for the opposing party who is now cross-examining if that party now wants to probe into bases that were otherwise inadmissible (e.g. the chart), that party may do so. IX. Authentication of Evidence 27

4.

A. Rule 901 Requirement of Authentication a. Evidence must be sufficient to support a finding. Conditional relevance under 104(b). b. A witness testimony will authenticate while under oath: - Do you recognize this object? - Yes. I do. - What is it? - Its my super duper model 2000 Turbo vibrator. - Authentication is now met, and the question is now one for the jury. c. Other methods of authentication (witness testimony is 901(b)(1) under the rule): i. Nonexpert opinion on handwriting; ii. Comparison by trier or expert witness; iii. Distinctive characteristics and the like iv. Voice identification may be through firsthand or mechanical transmission or recording. v. Telephone conversations; vi. Public records or reports; vii. Ancient documents or data compilation; viii. Process or system; ix. Methods provided by statute or rule. d. If something is not automatically recognizable (e.g. a pair of scissors), how does one differentiate? i. Chain of custody: method by which you assure, by the time the evidence gets to court, that the thing kept in custody all this time is the same thing. ii. An officer will likely testify to authentication.

28

You might also like